Équation diophantienne

Bonjour

Pour $a>81$, montrer qu'il existe $x,y,z$ entiers strictement positifs tels que
$x^3+y^3=a(a^3+z^3)$
Par quoi commencer ? Factoriser avec identités remarquables ?
Merci.

Réponses

  • Voici un début51677
  • une idée : tu peux peut être partir sur la résolution d'équation du 3ième degré .51683
  • La résolution d une équation de degré 3 n aidera pas pour
    les solutions entières, il me semble.
  • on peut poser a=u+v et z=u-v et x=f+g et y=f-g non?
  • Je propose la solution $(x,y,z)=(-2016,2016,-a)$.

    Plus sérieusement, peux-tu préciser :

    1) Où doivent vivre $x$, $y$ et $z$ ?

    2) L'origine de cette question ?
  • Rectification x,y,z entiers strictement positifs.
  • on peut déjà diviser par a^3
  • Bonjour @etanche,

    Est-ce un énoncé ou une question que tu te poses ?

    Ne doit-on pas supposer que $a$ est un entier carré ? La condition $\displaystyle a>81=9^2$ est suspecte à mes yeux.
  • l'équation est équivalente à x^3+y^3=a(z^3+1) en divisant par a^3
    maintenant on pose x=u+v et y=u-v et z=f+g et a=f-g
    En injectant cela dans l'équation cela donne :
    2u(u²+3v²)=(f-g)(f+g+1)((f+g)²-(f+g)+1)
  • C'est rigolo, ce truc. Voilà ce que j'ai fini par trouver :

    On peut vérifier que si $t$ est un entier $\geqslant3$, alors $3(t+1)^3<9t^3$. Par conséquent $]\!]81,+\infty[\![=\displaystyle\bigcup_{t\geqslant3}\;]\!]3t^3,9t^3[\![$.
    Il existe donc un entier $t\geqslant3$ tel que $3t^3<a<9t^3$ et $(x,y,z)=(9t^4,3ta-9t^4,9t^3-a)$ est alors une solution.
  • Bonjour @uvdose, Bravo (tu)(tu).
  • j'ai autre chose aussi si on pose a=b^3
    avec l'équation équivalente x^(3)+y^(3)=a(z^(3)+1) cela donne x^(3)+y^(3)=(bz)^(3)+b^(3) de la forme u^(3)+v^(3)=t^(3)+v^(3)
    ce qui est facilement résoluble http://www.math.harvard.edu/~elkies/4cubes.html http://edouardlucas.free.fr/pdf/oeuvres/analyse_indeterminee.pdf
  • je donnerai les solutions paramétriques ce soir elles sont dans le livre de Hardy que je n'ai pas la (:P) mais en tous cas la solution d'uvdose est élégante!
  • @max8128 : bien sûr, si tu supposes que $a=b^3$, l'équation possède la solution évidente $(x,y,z)=(b^4,b,1)$ !
  • Intéressant, certes... mais en quoi pour l'exercice initial ?
  • si on a est un cube alors l'équation est de la forme x^(3)+y^(3)+z^(3)=k
  • Relis ce que j'ai écrit plus haut : le cas où $a$ est un cube est trivial.
  • prenons a=1 alors 1=x^(3)+y^(3)+z^(3) avec z négatif
    on a la solution non triviale51717
  • Le cas $a=1$ est très particulier... Sans compter que l'énoncé impose $a>81$...

    Ceci dit, en y regardant de plus près, en partant de la formule que tu donnes, il doit y avoir moyen de redécouvrir celle que j'ai donnée plus haut (parce qu'en effet, en prenant $a=1$, on tombe sur la même chose). Il est probable que le concepteur du problème initial soit parti de cette formule...
  • Pour uvdose 22/20 et max8128 22/20
  • Bonjour voici une autre paramétrisation dans le cas ou l’équation de départ est équivalente à x^(3)+y^(3)=u^(3)+v^(3)

    x=c(1-(a-3b)(a²+3b²)) u=c((a+3b)-(a²+3b²)²)
    y=c((a+3b)(a²+3b²)-1) v=c((a²+3b²)²-(a-3b))
  • Pour trouver le lien il faut poser a=p et b=q et p=3q et t=-2q et on retombe sur le résultat de uvdose pour a=1 pour la suite c'est à dire pour d'autre valeurs de a je cale un peu ...
  • voici la solution complète (de Piezas): si ce n'est pas assez lisible il y a le site https://sites.google.com/site/tpiezas/01051763
  • Bonjour à tous

    Bravo uvdose pour ce joli résultat. Cela a déjà été dit plusieurs fois mais il ne faut pas bouder son plaisir !

    J'ai cherché des solutions sous la forme
    \[\begin{cases}
    x=a_1\,z+a_2 \\
    y=b_1\,z+b_2
    \end{cases}\]
    et je suis tombé sur la même solution (qui semble unique sous cette forme). Je dois cependant avouer que cette forme est "fortement inspirée" de ton résultat ! :-D As-tu pratiqué de la sorte ou différemment ?

    D'autre part j'ai effectué une recherche systématique par force brute : on fixe une valeur maximum pour $z$ et on cherche tous les couples $(x,y)$. Pour $a$ donné, il y plusieurs solutions. Par exemple pour $a=82$ et $z<10\,000$, j'ai trouvé

    \[\begin{array}{|c|c|c|}
    \hline
    z & x & y \\
    \hline
    86 & 32 & 460 \\
    \hline
    161 & 9 & 729 \\
    \hline
    431 & 341 & 1873 \\
    \hline
    918 & 1900 & 3840 \\
    \hline
    984 & 3034 & 3690 \\
    \hline
    1548 & 5122 & 5538 \\
    \hline
    1794 & 1405 & 7779 \\
    \hline
    7952 & 17774 & 32902 \\
    \hline
    \end{array}\]

    On remarque que pour $z=161$ on a ta solution avec $t=3$ (en échangeant $x$ et $y$) et que pour $z=984$ on a $x=a\,X,y=a\,Y,z=a_,Z$ avec $X^3+Y^3=a(1+Z^3)$, équation proposée par max8128. Reste à en trouver une paramétrisation.

    @max8128 : super ce site, mais c'est loin d'être fini, car les solutions sont données dans $\Q$ et il n'y a pas de lien entre $n$ et $c$. Pour autant, cela nous donne de la matière à moudre et c'est bien l'essentiel !
  • j'ai une idée barbare en tête je vous la présente :-D:
    on part de ce qu'un nombre entier quelconque puisse s'écrire sous la forme suivante a=a'^3+...+e^3 (5 termes cubiques)
    on injecte ça dans l’équation équivalente :

    x^3+y^3=(a'^3+...+e^3)(z^3+1)

    et on est ramener à un problème de Tarry-Escott http://arxiv.org/pdf/1603.00206.pdf . http://euler.free.fr/eslp/k1to9.htm
    http://www.oocities.org/titus_piezas/RamCube.htm
    Avec ici j=3 et n=651789
  • Plus sérieusement pour trouver une paramétrisation il suffit de poser u^(3)+v^(3)=1+z^(3) et de remplacer cela dans l'équation équivalente ce qui donne a(u^(3)+v^(3))=x^(3)+y^(3) et on retrouve :



    a^3+b^3 + n(c^3+d^3) = 0



    a = (1 - mn(p-3q))r

    b = (-1 + mn(p+3q))r

    c = (m^2*n - (p+3q))r

    d = (-m^2*n + (p-3q))r

    m = p^2+3q^2

    il reste a traiter le cas ou 1+z^(3) est différent de la somme de 2 cubes

    de plus j'ai trouvé ceci : (1+9t^3+648t^6-3888t^9)^3 + (-135t^4+3888t^10)^3 + (-3t-81t^4+1296t^7-3888t^10)^3 = 1 ce qui est différent bien sur de la paramétrisation donnée par uvdose.
  • Bonjour serge17 ! Avec l'équation sous la forme $(x+y)(x^2-xy+y^2)=a(a+z)(a^2-az+z^2)$, l'idée de départ était d'essayer de trouver une solution $(x,y,z)$ avec $x+y$ sous la forme $k\times a$ et avec $a+z$ sous la forme $u\times k$.

    @etanche : j'aimerais toujours savoir quelle est l'origine de ta question... un problème d'olympiade ?
  • uvdose
    Transmit par un étudiant de Paris 6 exo d'une feuille de TD

    [Inutile de recopier le message précédent. AD]
  • en posant az^3=x²

    cela donne x^3+y^3=a^4+x²

    dont la solution est sur ce site page 22 http://www.ucd.ie/t4cms/Diophantine Equations 12.03.16.pdf

    mais existe t-il une solution générale ? (sauf celle de uvdose)
  • Bonsoir, j'ai trouvé ceci d’intéressant :51877
    51881
    51883
  • on sait que tout entier s'écrit sous la forme de 5 cubes
    1)on divise par a^3
    2)on pose a=a31+...+a35
    3)on injecte dans l'équation
    4)on obtient une éqution de la forme x31+...+x36=y31+...+y36
    5)on utilise la solution de choudrhy http://arxiv.org/pdf/1603.00206.pdf51905
    51907
  • Voici une autre paramétrisation

    On part de l'équation x3+y3+z3=ht3

    la paramétrisation de cette équation est la suivante .
    x=27h3-81h²k3+45hk6+k9
    y=-27h3+81h²k3+27hk6-k9
    z=108h²k3-36hk6
    t=6(3hk+k4

    on pose -z3=h4 cela donne :


    -z3=h4=-(108h²k3-36hk6)3

    D'où h=k9(108h-3k²)3

    et on réinjecte dans l'équation
Connectez-vous ou Inscrivez-vous pour répondre.